Help putting differential equations into matrix form

Click For Summary
To convert the given system of inhomogeneous differential equations into matrix form, the equations can be expressed as a matrix equation involving the state vector and a function vector. The correct matrix representation is: [dx/dt, dy/dt] = [5, -1; 1, 3] * [x; y] + [2; -4t]. The initial conditions x(0)=1 and y(0)=2 can be applied later to find the specific solution. The discussion highlights the importance of ensuring the matrix is square and correctly formatted to include derivatives. Proper understanding of the matrix form is crucial for solving the system effectively.
hajjar0415
Messages
2
Reaction score
0

Homework Statement



Hello, I am trying to put the following equations into matrix form in order to solve the system. If anyone could please explain to me how to do it or show me an example it would be awesome.

All material given in question:

For the system of inhomogeneous differential equations,
dx/dt = 5x-y+2

dy/dt = x + 3y – 4t

and the initial condition, x(0)=1 and y(0)=2.

a)Arrange the system into matrix form
b)Find the diagonal or Jordan form of the system matrix
c)Write the general solution in the form of the matrix exponential
d)Use the initial condition to find the solution x(t) and y(t)

Thanks for any help

Homework Equations





The Attempt at a Solution



I am aware that the matrix has to be square in order to proceed.

What i have so far is:

[5 -1 0 ] [x] + [2]
[1 3 -4 ] [y] + [0]
[0 0 0 ] [t] + [0]

The -4t is what is throwing me off because there is no dt/dt equation given so i have put all 0's in the matrix.
 
Physics news on Phys.org
##t## is your independent variable. Write your system in the form$$
\begin{bmatrix} x'\\y'
\end{bmatrix}=
\begin{bmatrix}
a&b\\c&d
\end{bmatrix}
\begin{bmatrix}
x\\y
\end{bmatrix}+
\begin{bmatrix}
f(t)\\g(t)
\end{bmatrix}$$
 
So would the matrix form be:

5 -1 , x + 2
1 3 ,y + -4t

thanks for the help
 
hajjar0415 said:
So would the matrix form be:

5 -1 , x + 2
1 3 ,y + -4t

thanks for the help

I showed you one form. But yours isn't like mine because you have no derivatives nor equals signs. Surely your text shows you what form to use.
 
Question: A clock's minute hand has length 4 and its hour hand has length 3. What is the distance between the tips at the moment when it is increasing most rapidly?(Putnam Exam Question) Answer: Making assumption that both the hands moves at constant angular velocities, the answer is ## \sqrt{7} .## But don't you think this assumption is somewhat doubtful and wrong?

Similar threads

  • · Replies 3 ·
Replies
3
Views
2K
Replies
3
Views
2K
  • · Replies 5 ·
Replies
5
Views
2K
Replies
3
Views
1K
Replies
8
Views
3K
  • · Replies 6 ·
Replies
6
Views
1K
  • · Replies 4 ·
Replies
4
Views
1K
Replies
7
Views
2K
  • · Replies 7 ·
Replies
7
Views
2K
  • · Replies 3 ·
Replies
3
Views
1K